$intfrac{sin(x)}{sqrt{1-cos^2(x)}}$ Where did I mess up the domain?












1












$begingroup$


So we have $$intfrac{sin(x)}{sqrt{1-cos^2(x)}}dx$$
My book uses a $u$-substitution with $u=cos(x)$, $du=-sin(x)$,and they get $$intfrac{-du}{sqrt{1-u^2}}$$ which gives them $arccos(u)+C=arccos(cos(x))+C$



I understand that the domain of $arccos(x)$ means that $arccos(cos(x))$ is only equal to $x$ on $[0,pi]$, and is equal to some shifted version of $x$ for other values. However, I solved the integral differently and it resulted in $x + C$, but that isn't correct. I know that somewhere I messed up the domain, but I can't tell where. Here is my solution.
$$intfrac{sin(x)}{sqrt{1-cos^2(x)}}dx$$
$1-cos^2(x)=sin^2(x)$ so $$intfrac{sin(x)}{sqrt{1-cos^2(x)}}dx=intfrac{sin(x)}{sqrt{sin^2(x)}}=intfrac{sin(x)}{sin(x)}dx=int{1cdot dx}$$
So I get $x + C$ as my answer. Where did I mess up?










share|cite|improve this question











$endgroup$












  • $begingroup$
    What makes you think that it is wrong ? And why do you incriminate the domain ?
    $endgroup$
    – Yves Daoust
    Jan 29 at 12:56


















1












$begingroup$


So we have $$intfrac{sin(x)}{sqrt{1-cos^2(x)}}dx$$
My book uses a $u$-substitution with $u=cos(x)$, $du=-sin(x)$,and they get $$intfrac{-du}{sqrt{1-u^2}}$$ which gives them $arccos(u)+C=arccos(cos(x))+C$



I understand that the domain of $arccos(x)$ means that $arccos(cos(x))$ is only equal to $x$ on $[0,pi]$, and is equal to some shifted version of $x$ for other values. However, I solved the integral differently and it resulted in $x + C$, but that isn't correct. I know that somewhere I messed up the domain, but I can't tell where. Here is my solution.
$$intfrac{sin(x)}{sqrt{1-cos^2(x)}}dx$$
$1-cos^2(x)=sin^2(x)$ so $$intfrac{sin(x)}{sqrt{1-cos^2(x)}}dx=intfrac{sin(x)}{sqrt{sin^2(x)}}=intfrac{sin(x)}{sin(x)}dx=int{1cdot dx}$$
So I get $x + C$ as my answer. Where did I mess up?










share|cite|improve this question











$endgroup$












  • $begingroup$
    What makes you think that it is wrong ? And why do you incriminate the domain ?
    $endgroup$
    – Yves Daoust
    Jan 29 at 12:56
















1












1








1


0



$begingroup$


So we have $$intfrac{sin(x)}{sqrt{1-cos^2(x)}}dx$$
My book uses a $u$-substitution with $u=cos(x)$, $du=-sin(x)$,and they get $$intfrac{-du}{sqrt{1-u^2}}$$ which gives them $arccos(u)+C=arccos(cos(x))+C$



I understand that the domain of $arccos(x)$ means that $arccos(cos(x))$ is only equal to $x$ on $[0,pi]$, and is equal to some shifted version of $x$ for other values. However, I solved the integral differently and it resulted in $x + C$, but that isn't correct. I know that somewhere I messed up the domain, but I can't tell where. Here is my solution.
$$intfrac{sin(x)}{sqrt{1-cos^2(x)}}dx$$
$1-cos^2(x)=sin^2(x)$ so $$intfrac{sin(x)}{sqrt{1-cos^2(x)}}dx=intfrac{sin(x)}{sqrt{sin^2(x)}}=intfrac{sin(x)}{sin(x)}dx=int{1cdot dx}$$
So I get $x + C$ as my answer. Where did I mess up?










share|cite|improve this question











$endgroup$




So we have $$intfrac{sin(x)}{sqrt{1-cos^2(x)}}dx$$
My book uses a $u$-substitution with $u=cos(x)$, $du=-sin(x)$,and they get $$intfrac{-du}{sqrt{1-u^2}}$$ which gives them $arccos(u)+C=arccos(cos(x))+C$



I understand that the domain of $arccos(x)$ means that $arccos(cos(x))$ is only equal to $x$ on $[0,pi]$, and is equal to some shifted version of $x$ for other values. However, I solved the integral differently and it resulted in $x + C$, but that isn't correct. I know that somewhere I messed up the domain, but I can't tell where. Here is my solution.
$$intfrac{sin(x)}{sqrt{1-cos^2(x)}}dx$$
$1-cos^2(x)=sin^2(x)$ so $$intfrac{sin(x)}{sqrt{1-cos^2(x)}}dx=intfrac{sin(x)}{sqrt{sin^2(x)}}=intfrac{sin(x)}{sin(x)}dx=int{1cdot dx}$$
So I get $x + C$ as my answer. Where did I mess up?







indefinite-integrals trigonometric-integrals






share|cite|improve this question















share|cite|improve this question













share|cite|improve this question




share|cite|improve this question








edited Jan 29 at 13:00









Thomas Shelby

4,5862727




4,5862727










asked Jan 29 at 12:49









Ibrahim ElsadekIbrahim Elsadek

112




112












  • $begingroup$
    What makes you think that it is wrong ? And why do you incriminate the domain ?
    $endgroup$
    – Yves Daoust
    Jan 29 at 12:56




















  • $begingroup$
    What makes you think that it is wrong ? And why do you incriminate the domain ?
    $endgroup$
    – Yves Daoust
    Jan 29 at 12:56


















$begingroup$
What makes you think that it is wrong ? And why do you incriminate the domain ?
$endgroup$
– Yves Daoust
Jan 29 at 12:56






$begingroup$
What makes you think that it is wrong ? And why do you incriminate the domain ?
$endgroup$
– Yves Daoust
Jan 29 at 12:56












3 Answers
3






active

oldest

votes


















2












$begingroup$

You messed up because $sqrt{sin^2(x)} =sin x$ is not true for all values of $x$. In fact, it is only true if $sin(x)geq0$. If $sin(x)<0$, then $sqrt{sin^2(x)}=-sin(x)$. In general, $sqrt{sin^2(x)} = |sin x|$.






share|cite|improve this answer









$endgroup$













  • $begingroup$
    OK, so if i were to do $intfrac{sinx}{|{sinx}|}dx$ that would be right? I guess I have to do u-sub because I don't know how to integrate absolute values. Thanks
    $endgroup$
    – Ibrahim Elsadek
    Jan 29 at 13:02





















2












$begingroup$

We have $frac{sin(x)}{sqrt{sin^2(x)}}=frac{sin(x)}{|sin(x)|}=1 iff sin(x) >0$



and $frac{sin(x)}{sqrt{sin^2(x)}}=frac{sin(x)}{|sin(x)|}=-1 iff sin(x) <0$.



Can you take it from here ?






share|cite|improve this answer









$endgroup$





















    1












    $begingroup$

    The integrand is a square wave. Its antiderivative is a triangle wave.



    enter image description here






    share|cite|improve this answer









    $endgroup$














      Your Answer





      StackExchange.ifUsing("editor", function () {
      return StackExchange.using("mathjaxEditing", function () {
      StackExchange.MarkdownEditor.creationCallbacks.add(function (editor, postfix) {
      StackExchange.mathjaxEditing.prepareWmdForMathJax(editor, postfix, [["$", "$"], ["\\(","\\)"]]);
      });
      });
      }, "mathjax-editing");

      StackExchange.ready(function() {
      var channelOptions = {
      tags: "".split(" "),
      id: "69"
      };
      initTagRenderer("".split(" "), "".split(" "), channelOptions);

      StackExchange.using("externalEditor", function() {
      // Have to fire editor after snippets, if snippets enabled
      if (StackExchange.settings.snippets.snippetsEnabled) {
      StackExchange.using("snippets", function() {
      createEditor();
      });
      }
      else {
      createEditor();
      }
      });

      function createEditor() {
      StackExchange.prepareEditor({
      heartbeatType: 'answer',
      autoActivateHeartbeat: false,
      convertImagesToLinks: true,
      noModals: true,
      showLowRepImageUploadWarning: true,
      reputationToPostImages: 10,
      bindNavPrevention: true,
      postfix: "",
      imageUploader: {
      brandingHtml: "Powered by u003ca class="icon-imgur-white" href="https://imgur.com/"u003eu003c/au003e",
      contentPolicyHtml: "User contributions licensed under u003ca href="https://creativecommons.org/licenses/by-sa/3.0/"u003ecc by-sa 3.0 with attribution requiredu003c/au003e u003ca href="https://stackoverflow.com/legal/content-policy"u003e(content policy)u003c/au003e",
      allowUrls: true
      },
      noCode: true, onDemand: true,
      discardSelector: ".discard-answer"
      ,immediatelyShowMarkdownHelp:true
      });


      }
      });














      draft saved

      draft discarded


















      StackExchange.ready(
      function () {
      StackExchange.openid.initPostLogin('.new-post-login', 'https%3a%2f%2fmath.stackexchange.com%2fquestions%2f3092128%2fint-frac-sinx-sqrt1-cos2x-where-did-i-mess-up-the-domain%23new-answer', 'question_page');
      }
      );

      Post as a guest















      Required, but never shown

























      3 Answers
      3






      active

      oldest

      votes








      3 Answers
      3






      active

      oldest

      votes









      active

      oldest

      votes






      active

      oldest

      votes









      2












      $begingroup$

      You messed up because $sqrt{sin^2(x)} =sin x$ is not true for all values of $x$. In fact, it is only true if $sin(x)geq0$. If $sin(x)<0$, then $sqrt{sin^2(x)}=-sin(x)$. In general, $sqrt{sin^2(x)} = |sin x|$.






      share|cite|improve this answer









      $endgroup$













      • $begingroup$
        OK, so if i were to do $intfrac{sinx}{|{sinx}|}dx$ that would be right? I guess I have to do u-sub because I don't know how to integrate absolute values. Thanks
        $endgroup$
        – Ibrahim Elsadek
        Jan 29 at 13:02


















      2












      $begingroup$

      You messed up because $sqrt{sin^2(x)} =sin x$ is not true for all values of $x$. In fact, it is only true if $sin(x)geq0$. If $sin(x)<0$, then $sqrt{sin^2(x)}=-sin(x)$. In general, $sqrt{sin^2(x)} = |sin x|$.






      share|cite|improve this answer









      $endgroup$













      • $begingroup$
        OK, so if i were to do $intfrac{sinx}{|{sinx}|}dx$ that would be right? I guess I have to do u-sub because I don't know how to integrate absolute values. Thanks
        $endgroup$
        – Ibrahim Elsadek
        Jan 29 at 13:02
















      2












      2








      2





      $begingroup$

      You messed up because $sqrt{sin^2(x)} =sin x$ is not true for all values of $x$. In fact, it is only true if $sin(x)geq0$. If $sin(x)<0$, then $sqrt{sin^2(x)}=-sin(x)$. In general, $sqrt{sin^2(x)} = |sin x|$.






      share|cite|improve this answer









      $endgroup$



      You messed up because $sqrt{sin^2(x)} =sin x$ is not true for all values of $x$. In fact, it is only true if $sin(x)geq0$. If $sin(x)<0$, then $sqrt{sin^2(x)}=-sin(x)$. In general, $sqrt{sin^2(x)} = |sin x|$.







      share|cite|improve this answer












      share|cite|improve this answer



      share|cite|improve this answer










      answered Jan 29 at 12:53









      5xum5xum

      91.9k394161




      91.9k394161












      • $begingroup$
        OK, so if i were to do $intfrac{sinx}{|{sinx}|}dx$ that would be right? I guess I have to do u-sub because I don't know how to integrate absolute values. Thanks
        $endgroup$
        – Ibrahim Elsadek
        Jan 29 at 13:02




















      • $begingroup$
        OK, so if i were to do $intfrac{sinx}{|{sinx}|}dx$ that would be right? I guess I have to do u-sub because I don't know how to integrate absolute values. Thanks
        $endgroup$
        – Ibrahim Elsadek
        Jan 29 at 13:02


















      $begingroup$
      OK, so if i were to do $intfrac{sinx}{|{sinx}|}dx$ that would be right? I guess I have to do u-sub because I don't know how to integrate absolute values. Thanks
      $endgroup$
      – Ibrahim Elsadek
      Jan 29 at 13:02






      $begingroup$
      OK, so if i were to do $intfrac{sinx}{|{sinx}|}dx$ that would be right? I guess I have to do u-sub because I don't know how to integrate absolute values. Thanks
      $endgroup$
      – Ibrahim Elsadek
      Jan 29 at 13:02













      2












      $begingroup$

      We have $frac{sin(x)}{sqrt{sin^2(x)}}=frac{sin(x)}{|sin(x)|}=1 iff sin(x) >0$



      and $frac{sin(x)}{sqrt{sin^2(x)}}=frac{sin(x)}{|sin(x)|}=-1 iff sin(x) <0$.



      Can you take it from here ?






      share|cite|improve this answer









      $endgroup$


















        2












        $begingroup$

        We have $frac{sin(x)}{sqrt{sin^2(x)}}=frac{sin(x)}{|sin(x)|}=1 iff sin(x) >0$



        and $frac{sin(x)}{sqrt{sin^2(x)}}=frac{sin(x)}{|sin(x)|}=-1 iff sin(x) <0$.



        Can you take it from here ?






        share|cite|improve this answer









        $endgroup$
















          2












          2








          2





          $begingroup$

          We have $frac{sin(x)}{sqrt{sin^2(x)}}=frac{sin(x)}{|sin(x)|}=1 iff sin(x) >0$



          and $frac{sin(x)}{sqrt{sin^2(x)}}=frac{sin(x)}{|sin(x)|}=-1 iff sin(x) <0$.



          Can you take it from here ?






          share|cite|improve this answer









          $endgroup$



          We have $frac{sin(x)}{sqrt{sin^2(x)}}=frac{sin(x)}{|sin(x)|}=1 iff sin(x) >0$



          and $frac{sin(x)}{sqrt{sin^2(x)}}=frac{sin(x)}{|sin(x)|}=-1 iff sin(x) <0$.



          Can you take it from here ?







          share|cite|improve this answer












          share|cite|improve this answer



          share|cite|improve this answer










          answered Jan 29 at 12:54









          FredFred

          48.9k11849




          48.9k11849























              1












              $begingroup$

              The integrand is a square wave. Its antiderivative is a triangle wave.



              enter image description here






              share|cite|improve this answer









              $endgroup$


















                1












                $begingroup$

                The integrand is a square wave. Its antiderivative is a triangle wave.



                enter image description here






                share|cite|improve this answer









                $endgroup$
















                  1












                  1








                  1





                  $begingroup$

                  The integrand is a square wave. Its antiderivative is a triangle wave.



                  enter image description here






                  share|cite|improve this answer









                  $endgroup$



                  The integrand is a square wave. Its antiderivative is a triangle wave.



                  enter image description here







                  share|cite|improve this answer












                  share|cite|improve this answer



                  share|cite|improve this answer










                  answered Jan 29 at 13:03









                  Yves DaoustYves Daoust

                  131k676229




                  131k676229






























                      draft saved

                      draft discarded




















































                      Thanks for contributing an answer to Mathematics Stack Exchange!


                      • Please be sure to answer the question. Provide details and share your research!

                      But avoid



                      • Asking for help, clarification, or responding to other answers.

                      • Making statements based on opinion; back them up with references or personal experience.


                      Use MathJax to format equations. MathJax reference.


                      To learn more, see our tips on writing great answers.




                      draft saved


                      draft discarded














                      StackExchange.ready(
                      function () {
                      StackExchange.openid.initPostLogin('.new-post-login', 'https%3a%2f%2fmath.stackexchange.com%2fquestions%2f3092128%2fint-frac-sinx-sqrt1-cos2x-where-did-i-mess-up-the-domain%23new-answer', 'question_page');
                      }
                      );

                      Post as a guest















                      Required, but never shown





















































                      Required, but never shown














                      Required, but never shown












                      Required, but never shown







                      Required, but never shown

































                      Required, but never shown














                      Required, but never shown












                      Required, but never shown







                      Required, but never shown







                      Popular posts from this blog

                      Can a sorcerer learn a 5th-level spell early by creating spell slots using the Font of Magic feature?

                      Does disintegrating a polymorphed enemy still kill it after the 2018 errata?

                      A Topological Invariant for $pi_3(U(n))$